¿Es la ecuación de Dirac equivalente a la ecuación de Klein-Gordon para su componente zurdo?

La ecuación de Dirac

(0) ( i γ a a metro ) ψ = 0
está dado por un operador de primer orden que actúa sobre un espinor de Dirac, que es la suma directa de un espinor de mano izquierda y un espinor de mano derecha. El hecho de que sea de primer orden tiene sentido si queremos tratarla como la ecuación de Schrödinger para alguna partícula, pero si queremos tratarla como la ecuación de movimiento de un campo, sería bueno que fuera de segundo orden como la Ecuación de Klein-Gordon.

Podemos escribir las matrices gamma como:

γ a = ( σ a σ ¯ a )
y así la ecuación de Dirac se puede dividir en dos:
(1) i σ a a ψ R metro ψ L = 0 (2) i σ ¯ a a ψ L metro ψ R = 0
Entonces podemos sustituir ( 2 ) en ( 1 ) Llegar
i σ a a ( i σ ¯ b b ψ L ) metro 2 ψ L = 0
que simplifica a
(3) ( 2 + metro 2 ) ψ L = 0
la ecuación de Klein-Gordon para el componente de mano izquierda! Por el contrario, si empezáramos con ( 3 ) entonces podríamos tomar ( 2 ) como nuestra definición de ψ R y derivar ( 1 ) .

Me parece que ( 3 ) es mucho más simple que ( 0 ) . Por ejemplo, contar los grados de libertad es mucho más sencillo. Entonces, ¿por qué todos los libros de texto usan la ecuación de Klein-Gordon para campos escalares pero luego cambian a la ecuación de Dirac cuando hablan de espinores? ¿Las dos versiones de las ecuaciones hacen predicciones diferentes de alguna manera? (¿Quizás después de cuantificar?) Si es así, ¿en qué se diferencian?

EDITAR: Para poner lo que digo en otras palabras: ¿la teoría de campo de un espinor de Dirac que obedece a la ecuación de Dirac es equivalente a la teoría de campo de un espinor zurdo que obedece a la ecuación de KG?

¿Mmm? Todos los libros de texto mencionan esta relación entre Dirac y KG. Excepto el tuyo, parece.
@arivero Todos mencionan que la ecuación de Dirac implica la ecuación de Klein-Gordon (pero no necesariamente que KG implica a Dirac). De cualquier manera, luego continúan usando la ecuación de Dirac para todo, aunque todavía están usando la ecuación de KG para campos escalares (por supuesto, también podrían convertir la ecuación de KG en una ecuación de primer orden, pero nadie hace esto tampoco).
Nota: Su argumento no muestra equivalencia, solo muestra una implicación unidireccional.

Respuestas (1)

La ecuación de Dirac es más restrictiva que la ecuación de Klein-Gordon. Para cada solución de la ecuación de Dirac, sus componentes serán una solución de la ecuación de Klein-Gordon, pero lo contrario no es cierto: si forma un espinor cuyas componentes son soluciones de la ecuación de Klein-Gordon, es posible que no resuelva la ecuación de Klein-Gordon. Ecuación de Dirac.

Si comenzamos con la ecuación de Klein-Gordon para todo el espinor ψ

( 2 + metro 2 ) ψ = 0
la solucion es
ψ = tu ( pag ) mi i pag X o ψ = v ( pag ) mi + i pag X
donde tu respuesta v son espinores arbitrarios. Pero, ¿qué sucede si reemplazamos estas soluciones en la ecuación original de Dirac?
( 2 + metro 2 ) tu ( pag ) mi i pag X = mi i pag X ( γ m pag m metro ) tu ( pag ) = 0
( 2 + metro 2 ) v ( pag ) mi + i pag X = mi + i pag X ( γ m pag m + metro ) v ( pag ) = 0
tu y v ¡Ya no son arbitrarios! En cambio, deben obedecer la restricción más fuerte
( γ m pag m metro ) tu ( pag ) = 0 ( γ m pag m + metro ) v ( pag ) = 0

Si considera solo la ecuación de Klein-Gordon, está introduciendo "soluciones" adicionales que en realidad no resuelven la ecuación de Dirac.

¿Por qué pasó esto? Puede considerar la ecuación de Klein-Gordon como la versión "cuadrada" de la ecuación de Dirac. Y cuando elevas al cuadrado una ecuación, siempre obtienes estas desagradables soluciones falsas: si tienes, por ejemplo, la ecuación ( X 3 ) = 5 la solucion es X = 8 , pero si lo elevas al cuadrado ( X 3 ) 2 = 5 2 entonces tienes dos soluciones, X = 8 y X = 2 . La primera ecuación implica la segunda, pero lo contrario no es cierto.

¿Es posible relacionar la restricción más fuerte con la teoría de las representaciones del grupo de Lorentz?
@arivero Efectivamente! Estas restricciones son necesarias porque los componentes del espinor se mezclarán bajo una transformada de Lorentz y la ecuación de Dirac los une. Sin ellos, cada componente de ψ sería independiente en cada marco de referencia, por lo que el espinor sería "4 objetos en el ( 0 , 0 ) rep" en lugar de "1 objeto en el ( 1 / 2 , 0 ) ( 0 , 1 / 2 ) representante
@Bosoneando ¡No estoy sugiriendo aplicar la ecuación KG a todo el espinor de Dirac! Solo lo estoy aplicando a un solo espinor zurdo. Entonces podemos construir un espinor de Dirac definiendo el componente de la mano derecha por ψ R = i metro σ ¯ a a ψ L . Este espinor de Dirac obedecerá entonces la ecuación de Dirac. Para poner lo que digo en otras palabras: la teoría de campo de un espinor de Dirac que obedece a la ecuación de Dirac es la misma que la teoría de campo de un espinor zurdo que obedece a la ecuación de KG.
@OscarCunningham consideremos el metro = 0 caso ya que está libre de la mezcla de componentes para que podamos evitar confusiones. Entonces tienes esta ecuación de Weyl: σ m m ψ R = 0 . ¿Cómo es equivalente a la ecuación de KG? Para empezar, es de primer orden en derivadas del espacio-tiempo.